Difference between revisions of "2022 AMC 10A Problems/Problem 3"
MRENTHUSIASM (talk | contribs) (Created page with "==Problem== The sum of three numbers is <math>96.</math> The first number is <math>6</math> times the third number, and the third number is <math>40</math> less than the secon...") |
Sugar rush (talk | contribs) (Redirected page to 2022 AMC 12A Problems/Problem 2) (Tag: New redirect) |
||
Line 1: | Line 1: | ||
− | + | #redirect [[2022 AMC 12A Problems/Problem 2]] | |
− | |||
− | |||
− | |||
− | |||
− | |||
− | |||
− | |||
− | |||
− | |||
− | |||
− | |||
− | |||
− | |||
− | |||
− | |||
− | |||
− |
Revision as of 22:02, 11 November 2022
Redirect to: